Consumer advocate: Economists reason that price gouging-increasing the price of goods when no alternative seller is a...

Jaimee-Salgado on October 14, 2018

Please explain

I'm having trouble understanding why E is the correct answer. Can someone please break this down? I think the wording is throwing me off.

Replies
Create a free account to read and take part in forum discussions.

Already have an account? log in

JeremyG on May 17, 2019

Bump. Please explain.

AllisonL on September 15, 2019

Could someone please break this problem down?

Irina on September 16, 2019

@AllisonL,

Great question.

The consumer advocate tells us that economists claim that price gauging allocates goods to people whose willingness to pay more shows that they really need those goods, hence the economists' argument must assume that the amount one is willing to pay correlates with need. The consumer advocate then argues that this assumption is wrong, willingness to pay is not actually proportional to need because people have a limited amount of money no matter how much they need a particular good. Then the consumer advocate concludes that a price increase is not efficient, so the whole economist's reasoning is actually wrong.

The question asks us which of the following roles does this claim that willingness to pay is not proportional to need play. The point of this claim is to question the assumption made by the economist's argument.

Let's see what (E) is trying to say:

It denies a claim that the argument takes to be assumed in the reasoning that it rejects.

"It" is the consumer advocate's claim/ statement.
"a claim that the argument takes to be assumed" is just a lengthy way to say "assumption"
"in the reasoning that it rejects" refers to the economist's argument as the advocate rejects the argument as a whole.

So to paraphrase (E) says:

The consumer advocate's claim rejects/ denies an assumption in the economist's argument, which is exactly what happens here. (E) intentionally uses very confusing language, but once you break it down, it fits well with the structure of the argument.

Let me know if this makes sense and if you have any further questions.